Difference between revisions of "2021 AMC 10B Problems/Problem 7"

m
m (Colored the diagram so it makes things clearer.)
Line 6: Line 6:
 
==Solution 1 (Intuition by Graph)==
 
==Solution 1 (Intuition by Graph)==
 
<asy>
 
<asy>
/* diagram made by samrocksnature */
+
/* diagram made by samrocksnature, edited by MRENTHUSIASM */
 
pair A=(10,0);
 
pair A=(10,0);
 
pair B=(-10,0);
 
pair B=(-10,0);
 
draw(A--B);
 
draw(A--B);
draw(circle((0,-1),1));
+
filldraw(circle((0,7),7),yellow);
draw(circle((0,-3),3));
+
filldraw(circle((0,-5),5),yellow);
draw(circle((0,-5),5));
+
filldraw(circle((0,-3),3),white);
draw(circle((0,7),7));
+
filldraw(circle((0,-1),1),white);
 
dot((0,7));
 
dot((0,7));
 
draw((0,7)--(0,0));
 
draw((0,7)--(0,0));
Line 19: Line 19:
 
label("$\ell$",(-9,0),S);
 
label("$\ell$",(-9,0),S);
 
</asy>
 
</asy>
After a bit of wishful thinking and inspection, we find that the above configuration maximizes our area, which is <math>49 \pi + (25-9) \pi=65 \pi \rightarrow \boxed{\textbf{(D)}}</math>  
+
After a bit of wishful thinking and inspection, we find that the above configuration maximizes our area, which is <math>49 \pi + (25-9) \pi=\boxed{\textbf{(D) }65\pi}.</math>  
  
 
~ samrocksnature
 
~ samrocksnature

Revision as of 15:48, 20 July 2021

Problem

In a plane, four circles with radii $1,3,5,$ and $7$ are tangent to line $\ell$ at the same point $A,$ but they may be on either side of $\ell$. Region $S$ consists of all the points that lie inside exactly one of the four circles. What is the maximum possible area of region $S$?

$\textbf{(A) }24\pi \qquad \textbf{(B) }32\pi \qquad \textbf{(C) }64\pi \qquad \textbf{(D) }65\pi \qquad \textbf{(E) }84\pi$

Solution 1 (Intuition by Graph)

[asy] /* diagram made by samrocksnature, edited by MRENTHUSIASM */ pair A=(10,0); pair B=(-10,0); draw(A--B); filldraw(circle((0,7),7),yellow); filldraw(circle((0,-5),5),yellow); filldraw(circle((0,-3),3),white); filldraw(circle((0,-1),1),white); dot((0,7)); draw((0,7)--(0,0)); label("$7$",(0,3.5),E); label("$\ell$",(-9,0),S); [/asy] After a bit of wishful thinking and inspection, we find that the above configuration maximizes our area, which is $49 \pi + (25-9) \pi=\boxed{\textbf{(D) }65\pi}.$

~ samrocksnature

Solution 2 (Intuition in Words)

Suppose that line $\ell$ is horizontal, and each circle lies either north or south to $\ell.$ We construct the circles one by one:

  1. Without the loss of generality, we draw the circle with radius $7$ north to $\ell.$
  2. To maximize the area of the desired region, we draw the circle with radius $5$ south to $\ell$ by intuition.
  3. Now, we need to subtract the circle with radius $3$ at least. The optimal situation is that the circle with radius $3$ encompasses the circle with radius $1,$ in which we do not need to subtract more. That is, the two smallest circles are on the same side of $\ell,$ but can be on either side. The diagram in Solution 1 shows one possible configuration of the four circles.

Together, the answer is $\pi\cdot7^2+\pi\cdot5^2-\pi\cdot3^2=\boxed{\textbf{(D) }65\pi}.$

~MRENTHUSIASM

Video Solution by OmegaLearn (Area of Circles and Logic)

https://youtu.be/yPIFmrJvUxM

~ pi_is_3.14

Video Solution by TheBeautyofMath

https://youtu.be/GYpAm8v1h-U?t=206

~IceMatrix

Video Solution by Interstigation

https://youtu.be/DvpN56Ob6Zw?t=555

~Interstigation

See Also

2021 AMC 10B (ProblemsAnswer KeyResources)
Preceded by
Problem 6
Followed by
Problem 8
1 2 3 4 5 6 7 8 9 10 11 12 13 14 15 16 17 18 19 20 21 22 23 24 25
All AMC 10 Problems and Solutions

The problems on this page are copyrighted by the Mathematical Association of America's American Mathematics Competitions. AMC logo.png